Vous êtes sur la page 1sur 6

Final Exam for 03-62-194: Mathematics for Business

3:30 - 6:30 P.M., December 16, 2011


Solutions
1. (4 marks) A company charting its prots notices that the relationship between the
number of units sold x and the prot y is linear. If 200 units are sold the prot is
$3, 100. If 250 units are sold the prot is $6, 000. Give an equation for the prot y as a
function of the number of units x.
Solution. The slope is m = (6000 3100)/(250 200) = 58. When x = 200, y = 3100
so that 3100 = 58(200) +b giving b = 8500. An equation is y = 58x 8500.
2. (5 marks) A manufacturer of a DVD player has a xed cost of $9, 800 and variable costs
of $65 per unit. The company sells each DVD for $100. Let x be the number of units
produced and sold.
(a) (1 mark) Give an equation for the cost function C(x).
(b) (1 mark) Give an equation for the revenue function R(x).
(c) (1 mark) Give an equation for the prot function P(x).
(d) (2 marks) How many units must be sold to break even?
Solution. We have
(a) C(x) = 65x + 9800.
(b) R(x) = 100x.
(c) P(x) = 35x 9800.
(d) We solve P(x) = 0 to get x = 280.
3. (5 marks) The demand for a certain product is given by 2p + 5q = 200 and the supply
is given by p 2q = 10, where p is the market price in dollars, and q is the quantity
demanded or supplied.
(a) (4 marks) Find the equilibrium price and equilibrium quantity.
(b) (1 mark) What quantity is supplied and demanded if the price is $60?
Solution. We solve the two equations to get p = 50 and q = 20. When p = 60 the
demand is q = 16 and the supply is q = 25.
4. (7 marks) Solve the following system by creating the associated augmented matrix and
then reducing it to echelon form.
x 2y + 3z = 4
2x + 3y z = 1
x + 2y 3z = 6
Solution. We have the following sequence of row equivalent matrices
[A|b] =

1 2 3 4
2 3 1 1
1 2 3 6

R2 R2 2R1
R3 R3 R1

1 2 3 4
0 7 7 7
0 4 6 10

R2 R2/7

1 2 3 4
0 1 1 1
0 4 6 10

R1 R1 + 2R2
R3 R3 4R2

1 0 1 2
0 1 1 1
0 0 2 6

R3 R3/2

1 0 0 1
0 1 0 2
0 0 1 3

The answer is x = 1, y = 2, and z = 3.


5. (4 marks) An investor has $235, 000 invested in three income properties. The rst prop-
erty earns 12%, the second earns 10% and the third earns 8% per year. The investors
annual income from the three properties is $22, 500. The amount invested at 8% is twice
that invested at 12%.
Set up, but do not solve, the system of linear equations whose solution will
give the amount invested in each property. One mark will be given for a
well-written denition of the variables.
Solution. Let x, y and z be the amount of money, in dollars, invested in the rst,
second, and third properties, etc. The equations are
x +y +z = 235, 000
.12x +.10y +.08z = 22, 500
2x z = 0
Page 2
6. (6 marks) Two linear programming problems have the same bounded, non-empty feasible
region. The corner points of the region are (0, 0), (7, 0), (6, 5), (4, 5), and (0, 3).
(a) (3 marks) What is the maximum value of C = x 2y?
(b) (3 marks) What is the minimum value of Z = 5x 3y?
Solution.
Corner Point C = x 2y Z = 5x 3y
(0, 0) 0 0
(7, 0) 7 35
(6, 5) 16 15
(4, 5) 14 5
(0, 3) 6 9
The maximum value of C = x 2y is 0. The minimum value of Z = 5x 3y is 9.
7. (6 marks) Solve the following system graphically and clearly indicate the solution.
2x + 5y 20
6x + 5y 60
x 0
y 0
Solution. The corner points (0, 0), (0, 4), (10, 0) and (5, 6).
8. (7 marks) Your job is to make, at minimum cost, a mixture of nuts from almonds,
cashews, and peanuts for an upcoming party. The almonds cost $2.50/kg, the cashews
cost $4.00/kg and the peanuts cost $1.50/kg. There are some restrictions. The mixture
of nuts must weigh 10kg or more. The combined weight of almonds and cashews must
be greater than or equal to the weight of the peanuts. The weight of the cashews must
be 1kg or more. The weight of almonds must be less than or equal to the weight of the
cashews.
Set up, but do not solve the LP. Be sure to include a well written denition
of the variables, which is worth 1 of the seven marks. Solution.
Let A, C, and P be the number of kilograms of almonds, cashews, and peanuts to be
purchased. The linear programming problem is to
min 2.5A + 4C + 1.5P = z
subject to A + C + P 10
A + C P 0
C 1
A C 0
A 0
C 0
P 0
Page 3
9. (5 marks) Graph the quadratic function y = x
2
6x + 5. Give the coordinates of the
vertex, the y-intercept and the two x-intercepts. Clearly label these points on the graph.
Solution. The vertex is (3, 4), the y-intercept is (0, 5) and the x-intercepts are (1, 0)
and (5, 0).
10. (5 marks) What is the future value of $3, 500 invested at 3% compounded semi-annually
after 4 years?
Solution. The future value is FV = 3500(1 +
.03
2
)
4(2)
= 3500(1.015)
8
= 3942.72.
11. (5 marks) What is the future value of $1, 000 invested at 2% compounded continuously
after 5 years?
Solution. The future value is FV = 1000e
5(.02)
= 1000e
.1
= 1105.17
12. (5 marks) If interest is 4% compounded monthly, what is the present value of $500 three
years from now?
Solution. The present value is PV =
500
(1+
.04
12
)
3(12)
= 443.55.
13. (5 marks) If interest is 3.5% compounded continuously, what is the present value of
$10, 000 six years from now?
Solution. The present value is PV =
10000
e
.035(6)
= 8105.84.
14. (6 marks) An advertising agency has found that when it promotes a new product in a
market with a population of 210 000 the number of people who are aware of this product
t weeks after the ad campaign starts is given by
f(t) = 210, 000(1 e
.045t
).
(a) (2 marks) How many people are aware of the product after 2 weeks?
(b) (4 marks) How many weeks will it take until half the people in this market are
aware of the new product?
Solution. (a) We have f(2) = 210, 000(1e
.0452
) = 18074.45. The number is 18, 074.
(b) We solve 105, 000 = 210, 000(1 e
.045t
) to get t =
ln(.5)
.045
= 15.4. It takes just over
15 weeks.
15. (4 marks) An insurance company received 25 claims. It is known that 2 of the claims
are fraudulent. If two claims are chosen at random for investigation, nd the probability
that
(a) (2 marks) both fraudulent claims are investigated.
(b) ( 2 marks) at most one fraudulent claim is investigated.
Solution. There is one way to pick both fraudulent claims and there are 25C2 = 300
ways of picking two cases from twenty-ve. The probability is 1/300. For part (b) we
have probability 299/300.
Page 4
16. (4 marks) A product goes through two dierent kinds of inspection; one for appearance
and one for functionality. The probability that the product passes the appearance in-
spection is 0.45. The probability that it passes the appearance inspection and not the
functionality inspection is 0.3. What is the probability that it will pass the appearance
inspection and the functionality inspection?
Solution. P(A) = P(A B) + P(A not B). We have .45 = P(A B) + .3 so that
P(A B) = .15.
17. (5 marks) NACO Body Shops has found that 14% of the cars on the road need to be
painted and that 10% need major body work. If a car needs major body work, there is
a 20% chance that it also needs to be painted.
(a) (2 marks) What is the probability that a randomly chosen car needs to be painted
and it needs major body work?
(b) (3 marks) What is the probability that a randomly chosen car needs major body
work, given that it needs to be painted?
Solution. Let A be the event of needing paint and let B be the event of needing body
work. We have P(A) = .14, P(A|B) = .2 and P(B) = .1.
(a) Pr(A B) = Pr(A|B)Pr(B) = (.2)(.1) = .02.
(b) Pr(B|A) = Pr(A B)/Pr(A) = .02/.14 = .143.
18. (4 marks) An experiment has four outcomes with value and probabilities given in the
table.
i 1 2 3 4
Value x
i
4 2 8 3
Probability P(x
i
) .2 .1 .3 .4
(a) (2 marks) What is the expected value ?
(b) (3 marks) What is the standard deviation ?
Solution. The expected value is
= x
1
P(x
1
) +x
2
P(x
2
) +x
3
P(x
3
) +x
4
P(x
4
)
= 4.6
The variance is

2
= (x
1
)
2
P(x
1
) + (x
2
)
2
P(x
2
) + (x
3
)
2
P(x
3
) + (x
4
)
2
P(x
4
)
= 5.24.
The standard deviation is

5.24 2.29.
Page 5
19. (3 marks) A hospital buys thermometers in boxes of 1000. The probability that a
thermometer is defective is 0.01. If 20 thermometers are selected from one box, what is
the probability that exactly 2 are defective?
Solution. This is a binomial experiment with p = .01, q = .99, n = 20 and x = 2. The
probability is
P(2) = 20C2(.01)
2
(.99)
18
=
20!
18!2!
(.01)
2
(.99)
18
.0159.
Solution.
20. (5 marks) In a certain store, there is a 0.02 probability that the scanned price in the bar
code scanner will not match the advertised price. The cashier scans 2000 items. What
is the probability of 30 or more mismatches?
Solution. We use the normal approximation to the binomial distributions. The mean
is = 2 000(.02) = 40 and the standard deviation is =

2 000(.02)(.98) 6.26. The


probability of 30is approximated by
P(X 29.5) = 1 NORMDIST(29.5, 40, 6.26, 1).
Page 6

Vous aimerez peut-être aussi